Können wir ein Axiomatisches System physikalischer Gesetze konstruieren?

Wenn wir ein axiomatisches System physikalischer Gesetze konstruieren, die voneinander unabhängig sind, wie in Axiomen in der Mathematik, welche sollten sie sein? Kann es ein so endliches System physikalischer Gesetze geben, das jedes physikalische Phänomen erklären kann? Oder ist es unmöglich, ein solches endliches axiomatisches System in physikalischen Gesetzen zu haben?

Das in der Mathematik untersuchte System von grundlegenden Standardaxiomen ist im praktischen Sinne nicht endlich. Sogar die KP-Mengentheorie , die eher auf der berechenbaren Seite liegt (wenn Sie so wollen, physikalischer), beinhaltet Axiomschemata , z. B. Ersetzung .
Mathematik ist eine Schöpfung des menschlichen Geistes. In einer physikalischen Theorie versuchen wir, etwas nachzubilden, indem wir unsere eigene Sprache Mathematik verwenden, es ist die einzige Sprache, die wir kennen. Das bedeutet nicht, dass Mathematik die Grundlage der Natur ist. Physik muss mathematisch nicht streng sein.
Könnten Sie Ihre Meinung deutlich machen? @Nick Kidman
Die Frage der "Axiomatisierung der Physik" wurde auch als Hilberts sechstes Problem gestellt, siehe zB en.wikipedia.org/wiki/Hilbert%27s_sixth_problem
@KaziarafatAhmed: Meinung zu was? OP schrieb "solch ein endliches System", indem er die physikalischen Axiome, nach denen er sucht, mit grundlegenden mathematischen Axomen verglich. Und ich wies darauf hin, dass diese tatsächlich nicht besonders endlich sind.
Ja, es heißt mathematische Physik. zB Wightman, und so weiter.

Antworten (3)

Ja, es ist möglich. Wie J. Bell eloquent schrieb, erklärt die Quantenmechanik zusammen mit einer endlichen Grenz-QED die gesamte Chemie und fast alles in der Physik. Es wurde 1930 von Weyl und Dirac axiomatisiert.

Es gibt nur sechs Axiome, was sicherlich eine endliche Zahl ist. Fünf wären noch besser ... da die meisten Physiker nicht mehr an die wörtliche Wahrheit des sechsten Axioms glauben.

Es gibt notorische Probleme mit dieser Axiomatisierung, aber sie können sicherlich behoben werden, obwohl sich die Physiker darüber nicht einig sind, wie sie behoben werden können. Das Problem wurde am logischsten von Wigner und später von JS Bell in seinem „Against Measurement“ analysiert, ich habe eine urheberrechtsfreie Kopie unter http://www.chicuadro.es/BellAgainstMeasurement.pdf veröffentlicht . Das heißt, die ersten drei Axiome gelten für alle physikalischen Systeme, die zweiten drei Axiome gelten nur für Messungen, aber sicherlich werden Messungen von physikalischen Messgeräten durchgeführt .... leider gelten die Antworten der ersten drei Axiome für die Interaktion eines mikroskopischen Systems mit einem Messgerät unterscheiden sich von den Ergebnissen, die sich ergeben, wenn man die zweiten drei Axiome auf denselben physikalischen Aufbau anwendet. Nicht widersprüchlich, aber so unterschiedlich, dass man sich nicht darauf geeinigt hat, wie man sie vergleichen soll.

Die meisten Physiker sind jetzt der Meinung, dass die Messaxiome nur Annäherungen sind und aus den ersten drei Axiomen als Annäherungen ableitbar sein sollten. HS Green veröffentlichte unter (glaube ich) Schroedingers Einfluss in Dublin ein äußerst wichtiges Papier, das die Physik des Messvorgangs als Phasenübergang analysierte, und es gab auch neuere Arbeiten. Siehe zum Beispiel mein eigenes http://arxiv.org/abs/quant-ph/0507017 .

Die einzige verbleibende Schwierigkeit besteht darin, entweder den Begriff „Wahrscheinlichkeit“ so zu definieren, wie er in diesen Axiomen vorkommt, oder einige weitere Axiome zu formulieren, um ihn mit den anderen Axiomen zu verbinden. Für den Quantenfall wurde dies in der erwähnten Arbeit getan, und etwas Ähnliches kann im klassischen Fall getan werden.

Dies ist die Antwort eines Experimentators:

Ich glaube, dass ein axiomatisches Modell , Anmerkung „Modell“, der Natur gefunden werden kann, aber als Experimentator bin ich misstrauisch gegenüber Behauptungen, dass „wir jetzt die Physik abgeschlossen haben und nur noch Details aufgeräumt werden müssen“, was die Behauptung zuvor war Die Quantenmechanik erschütterte die Wissenschaft zu Beginn des zwanzigsten Jahrhunderts.

Man sollte offen sein für die Möglichkeit, dass die Axiome möglicherweise geändert werden müssen, wenn wir uns immer weiter in Experimente mit neuen Technologien vertiefen und immer mehr vom Kosmos verstehen. Sonst versteinert die Physik.

Keine vollständige.

Kurt Gödel bewies, dass dies nicht möglich war, indem er sein "Unvollständigkeitstheorem" bewies. Es stellt sich heraus, dass wir in jedem axiomatischen System (ob diese Axiome mit physikalischen Gesetzen zu tun haben oder nicht) entweder Konsistenz oder Vollständigkeit wählen müssen, aber nicht beides.

Grundsätzlich besagt das "Unvollständigkeitstheorem", dass jedes "berechenbare axiomatische System" die folgenden Eigenschaften hat (z. B. eines, das physikalische Gesetze enthält):

  1. Wenn das System vollständig ist, kann es nicht konsistent sein.
  2. Die Konsistenz der Axiome kann innerhalb des Systems nicht bewiesen werden

Als Folge von 1 kann kein konsistentes Axiomatiksystem vollständig sein (das von Ihnen beschriebene System wäre hoffentlich konsistent). Wenn Sie also wollen, dass Ihre physikalischen Gesetze intern konsistent sind, müssen Sie akzeptieren, dass es wahre (beobachtbare) physikalische Gesetze gibt, die nicht bewiesen werden können.

In Bezug auf unendliche Systeme gibt es zwei Arten, abzählbar und nicht abzählbar. Der Unvollständigkeitssatz hat sich auch für abzählbare unendliche Mengen als wahr erwiesen. Im Fall unendlicher Axiomatiksysteme, die sich mit physikalischen Gesetzen befassen, wären sie zählbar, da jedes Axiomatikgesetz in die Menge der natürlichen Zahlen abgebildet werden könnte. Auch hier gelten Gödels Schlussfolgerungen; Entweder wäre dieses System konsistent, aber unvollständig, oder vollständig, aber inkonsistent.

Anscheinend kommen wir nicht darum herum, dass es unbeweisbare Wahrheiten gibt. Gödel lieferte ein einfaches Beispiel.

Sei S die Aussage „Diese Aussage ist nicht beweisbar“.

Wenn S wahr ist, können wir es nicht beweisen, da es unbeweisbar ist. Wenn wir jedoch beweisen können, dass S wahr ist, ist die Aussage selbst widersprüchlich, also inkonsistent.

Beachten Sie aus der Antwort oben das beredte Zitat von J. Bell: "Die Quantenmechanik erklärt zusammen mit einer endlichen Cut-off-QED die gesamte Chemie und FAST ALLES in der Physik." Unglücklicherweise für Bell hat Gödel gezeigt, dass die Quantenmechanik, solange sie versucht, innerlich konsistent zu sein, immer nur "FAST ALLES" und nicht wirklich "ALLES" sein wird. Wenn die Quantenmechanik tatsächlich die Fähigkeit erreicht, alles zu erklären, zeigt uns Gödel, dass wir guten Grund haben, nach ihren Selbstwidersprüchen (Inkonsistenzen) zu suchen.

bei allem Respekt vor Gödel (ich kenne ihn aus der Mengenlehre, "die Menge aller Mengen ist offen"), die "diese Aussage ist unbeweisbar" geht Tausende von Jahren zurück auf das kretische Paradoxon: "Ein Kreter sagte, alle Kreter seien Lügner" . en.wikipedia.org/wiki/Epimenides_paradox . Paradoxien in der heutigen Zeit werden durch Metaebenen aufgelöst, und ich glaube nicht, dass Gödels Theorem durch eine Metaebene aufgelöst werden kann.
Könnte sein. Auch der Apostel Paulus war sich dieses Zitats offensichtlich bewusst [Titus 1:12]
Ich sollte auch hinzufügen, dass niemand bewiesen hat, dass das Unvollständigkeitstheorem nicht durch eine Metaebene gelöst werden kann. Ungeachtet dessen legt die bloße Tatsache, dass es so aussieht, nahe, dass der Satz selbst kein Paradoxon, sondern eine wahre Eigenschaft von Axiomatiksystemen ist.
@Joseph f.johnson - niemand hat argumentiert, dass Mathematik inkonsistent ist. Es ist konsequent. Es ist jedoch nicht vollständig im Sinne von Gödel. Der Satz von Gödel hat 2 mögliche Arten von Systemen; konsistente, aber unvollständige oder vollständige, aber inkonsistente. Da der Mathematik ständig neue Axiome hinzugefügt werden, ist sie eindeutig unvollständig und kann daher auch konsistent sein. Anscheinend verstehen Sie Gödels Theorem nicht.
Ok, in Bezug auf Gödels Theorem und seine Beziehung zur QM lautet die umformulierte Frage, was ein berechenbares System ausmacht. Kurz gesagt, ein berechenbares System kann als endlicher Automat (Operatoren und Operanden) dargestellt werden. QM kann als endlicher Automat dargestellt werden und ist somit ein berechenbares System. (Um QM-Berechnungen durchzuführen, müssen Grammatik und Syntax, Operatoren und Operanden klar verwendet werden. QM ist also axiomatisch und berechenbar, daher gilt das Unvollständigkeitstheorem.
@Joseph f.johnson, hast du die Frage gelesen? Er fragte, ob ein axiomatisches System konstruiert werden könne, das "jede Art von Phänomen erklären könnte. Die gegebene Antwort bestritt nur, dass ein solches System vollständig wäre.
Gentzens Konsistenzbeweis befasst sich nur mit einer vereinfachten Teilmenge der Mathematik, insbesondere dem Teil der Mathematik, der nicht bewiesen werden kann. Gentzens Beweis verwendet zum Beispiel auch transfinite Induktion, die sicherlich keine Arithmetik selbst verwendet (gemäß Gödels Behauptung könnte sich kein System selbst beweisen) und wurde von Tarski Angus Macintyre nicht als überzeugend angesehen („Mathematical Signifikanz der Beweistheorie“, Phil. Trans. Royal Soc. A 363 (2005), 2419–2435, S. 2426). Außerdem (sagt MacIntyre) war Konsistenz in Gentzens Arbeit überhaupt nicht das Hauptproblem.
Was Sie hier also tun, ist, ein Strohmann-Argument über meine Verwendung von Gödel aufzubauen und dagegen zu argumentieren. Was Gödel bewies, war, dass, wenn die formalisierte Arithmetik konsistent ist, eine bestimmte numerische Kodierung dieser Tatsache innerhalb dieser Theorie selbst ausdrückbar, aber nicht beweisbar ist. Sein Theorem schloss überzeugende Beweise der Konsistenz der Arithmetik nicht aus, die innerhalb der Arithmetik nicht formalisierbare Mittel verwenden. In Bezug auf Gentzens Beitrag ist die Konsistenz der Arithmetik immer noch nicht aus der Arithmetik selbst beweisbar. Ihre Kritik erkennt dies nicht fair oder objektiv an.
Ich sollte auch hinzufügen, dass Gentzens Arbeit allgemein als Beweis des Satzes von Gödel angesehen wird. Durch die Bereitstellung eines direkten Beweises für die Unbeweisbarkeit des Prinzips der transfiniten Induktion, das in seinem Konsistenzbeweis von 1936 verwendet wurde, lieferte Gentzen Gödel die Möglichkeit, eine unbeweisbare Formel der Arithmetik zu entdecken, die zeigt, dass die Arithmetik unvollständig ist. Gödel nahm Gentzens Beweis und benutzte ein Kodierverfahren, um eine unbeweisbare arithmetische Formel zu konstruieren.
Ich verstehe nicht, was Sie mit «QM ... ist ein berechenbares System» meinen. Meinst du dasselbe wie Prof. Shor, in seiner Antwort physical.stackexchange.com/a/73366/6432 auf eine verwandte Frage, dh dass jedes physikalische Phänomen mit beliebiger Genauigkeit berechnet werden kann? (Durch Simulation des entsprechenden physikalischen Modells).
Sorry, das hätte ich deutlicher definieren sollen. Mit „berechenbares System“ verwende ich die Church-Turing-These, die besagt, dass jede Funktion, die von einem Algorithmus berechenbar ist, eine berechenbare Funktion ist. Daher ist jedes System, das aus berechenbaren Funktionen besteht, ein berechenbares System. Obwohl zunächst skeptisch, argumentierte Gödel etwa 1946 für die Church-Turing-These! (Um zu sehen, wie sich QM als berechenbares System qualifiziert, siehe Werner Depauli-Schimanovich, Eckehart Köhler und Friedrich Stadlers 'Berechenbarkeit in der Quantenmechanik'. Im Grunde meine ich es genauso wie Gödel (und Turing & Church).
Ich nehme an, Sie stimmen zu, dass es anscheinend keine regelmäßigen Vorhersagen der Quantenmechanik gibt, die angesichts berechenbarer Daten nicht berechenbar sind?
Es gibt einen feinen Unterschied zwischen berechenbar , wie es in der Mathematik verwendet wird, und berechenbar , wie es in der Physik verwendet wird. Wenn ich den Unterschied verstehe, liegt es darin, dass in der Mathematik derselbe Algorithmus in der Lage sein muss, die Antwort mit jedem gewünschten Grad an Annäherung zu geben. Aber in der Physik ist es uns egal, ob für jeden anderen Fall ein anderer Algorithmus benötigt würde. Das ist wie der Unterschied zwischen Omega-Konsistenz und Konsistenz. Möglicherweise gibt es innerhalb des Systems keinen Algorithmus zum Erzeugen des neuen und unterschiedlichen Algorithmus, der für jeden Fall benötigt wird. Daher weiß ich nicht, wie ich Ihre Frage beantworten soll.
In einer richtig formulierten Theory of Everything werden Fragen wie «Existiere ich?» sollte unmöglich zu formulieren sein. Tatsächlich sollte das Pronomen «ich» keine Übersetzung in die Physik haben, noch sollte das Verb «existieren». (Kant: Dasein ist kein Prädikat.) Es ist kein Zufall, dass ihre Verwendung in Lehrbüchern eher selten ist.
Ihre Klarstellung lässt mich immer noch im Dunkeln, was Ihre Bedeutung betrifft. Ein Axiom ist keine Funktion. Ein axiomatisches System besteht aus primitiven Konzepten und Axiomen. Die Axiome sind sicherlich keine Funktionen. Die primitiven Konzepte könnten sein. Aber in, sagen wir, Hilberts Axiomatisierung der euklidischen Geometrie bin ich mir nicht sicher, ob ich mich an irgendwelche Funktionen erinnere. Und, um zum anderen Extrem zu kommen, Principia Mathematica hat Funktionen, aber sie sind doch sicher alle berechenbar? Verneinung usw.? Was das mit Konsistenz zu tun hat, ist mir immer noch ein Rätsel. Ich hätte gedacht, dass berechenbare Systeme einfacher sind als unberechenbare....
In den drei grundlegenden Axiomen der QM kommt als einzige Funktion die Exponentialfunktion des Hamiltonoperators vor. Wenn der Hilbert-Raum endlichdimensional ist, dann ist dies offensichtlich in beiden Richtungen berechenbar. Nehmen wir der Diskussion halber an, dass unendliche Dimensionen kein Problem darstellen. In anderen Kommentaren und Antworten haben Sie deutlich gemacht, dass Sie es für notwendig halten, auch die Axiome der Logik einzubeziehen, also werfen Sie PM ein, alle Funktionen sind immer noch berechenbar ... und was nun?
Es ist nicht klar, was Sie mit "ein Axiom ist keine Funktion" meinen. Wir sprechen auch nicht speziell von QM, sondern von dem zugrunde liegenden Axiomatiksystem. QM muss bestimmte Dinge voraussetzen, um funktionieren zu können, und es sind „diese bestimmten Dinge“, die sich in seinen Axiomen zeigen. Axiome können abgeleitet werden (also auch Schlussfolgerungen), aber Axiome sind nicht nur die Axiome der Logik. Sie sind spezifisch für das verwendete formale System. In der Mathematik gibt es Axiome (dh 1+0=1 ist immer wahr, man kann nicht durch 0 teilen usw.). Die Axiome der Physik sind fast immer mathematisch, fast immer Gleichungen (F=MA). Was ist "PM"?
"Es stellt sich heraus, dass wir in jedem axiomatischen System (ob diese Axiome mit physikalischen Gesetzen zu tun haben oder nicht) entweder Konsistenz oder Vollständigkeit wählen müssen, aber nicht beides." Das wirkliche geschlossene Feld ist vollständig und konsistent, also ist es entweder kein "axiomatisches System" oder was Sie sagen, ist falsch.